Mathcenter Forum  

Go Back   Mathcenter Forum > คณิตศาสตร์โอลิมปิก และอุดมศึกษา > ข้อสอบโอลิมปิก
สมัครสมาชิก คู่มือการใช้ รายชื่อสมาชิก ปฏิทิน ข้อความวันนี้

ตั้งหัวข้อใหม่ Reply
 
เครื่องมือของหัวข้อ ค้นหาในหัวข้อนี้
  #16  
Old 13 พฤษภาคม 2007, 21:48
Mathophile's Avatar
Mathophile Mathophile ไม่อยู่ในระบบ
กระบี่ไว
 
วันที่สมัครสมาชิก: 31 มีนาคม 2007
ข้อความ: 250
Mathophile is on a distinguished road
Default

ช่วยเอาข้อสอบวิธีทำมาลงให้ครับ (โจทย์ยาวมาก... )
--------------------
การแข่งขันคณิตศาสตร์โอลิมปิก สอวน. ครั้งที่ 4
วันที่ 4-9 พฤษภาคม 2550
ข้อสอบวันที่สอง
----------
1. จงหาฟังก์ชัน $f:R \rightarrow R$ ทั้งหมดซึ่ง
$$\sum_{i=1}^{2549}f(x_i+x_{i+1})+f(\sum_{i=1}^{2550}x_i)\leq \sum_{i=1}^{2550}f(2x_i)$$สำหรับทุกจำนวนจริง $x_1,x_2,...,x_{2550}$

2. นักเรียนหญิง $n$ คนและนักเรียนชาย $n$ คนในชั้น ม.1/1 อยู่ในงานปาร์ตี้ที่มีการเต้นรำแห่งหนึ่ง ในแต่ละเพลง จะมีนักเรียนชายหญิงจับคู่ขึ้นไปเต้นรำอย่างน้อยหนึ่งคู่ นักเรียนทุกคนที่ขึ้นไปเต้นรำจะได้รับพวงมาลัยคนละหนึ่งพวงเมื่อจบเพลงเสมอ ถ้ามีการเต้นรำทั้งหมด $m$ เพลง
จงแสดงว่าสำหรับทุกจำนวนนับ $k\leq n$ จะต้องมีกลุ่มนักเรียนที่ประกอบด้วยชาย $k$ คนและหญิง $n-k$ คนซึ่งได้รับพวงมาลัยรวมกันอย่างน้อย $m$ พวง

3. วงกลมสองวงตัดกันที่จุด $X$ และ $Y$ เส้นตรงที่ผ่านจุดศูนย์กลางของวงกลมทั้งสองตัดวงกลมแรกที่จุด $A$ และ $C$ และตัดวงกลมที่สองที่จุด $B$ และ $D$ โดยที่จุด $B$ อยู่บนส่วนของเส้นตรง $AC$ และจุด $C$ อยู่บนส่วนของเส้นตรง $BD$ คอร์ดร่วม $XY$ ตัด $BC$ ที่จุด $P$ ให้จุด $O$ เป็นจุดใด ๆ บน $XP$ ที่อยู่ระหว่าง $X$ กับ $P$ ต่อ $CO$ พบวงกลมแรกที่จุด $M$ และต่อ $BO$ พบวงกลมที่สองที่จุด $N$ ต่อ $AM$ และ $DN$ ออกไปพบกันที่ $Z$ จงพิสูจน์ว่า $X,Y$ และ $Z$ อยู่บนเส้นตรงเดียวกัน

4. จงหาจำนวนเฉพาะ $p$ ทั้งหมดที่ทำให้ $\displaystyle{\frac{2^{p-1}-1}{p}}$ เป็นกำลังสองสัมบูรณ์

5. ชั้น ม.1 มีนักเรียนชาย 229 คนและนักเรียนหญิง 271 คน แบ่งเป็น 10 ห้อง ห้องละ 50 คน นักเรียนแต่ละห้องมีเลขที่ 1 ถึง 50 ครูต้องการจัดทีมวิ่งผลัดหนึ่งทีม โดยมีนักเรียนหญิง 1 คนและชาย 3 คน หรือนักเรียนหญิง 3 คนและชาย 1 คน และมีเงื่อนไขเพิ่มเติมว่านักเรียนสี่คนนี้มาจากสองห้อง ห้องละสองคนที่มีเลขที่ตรงกัน (เช่น นักเรียนเลขที่ 2 และ 15 จากห้อง ม.1/1 และ ม.1/3)
จงแสดงว่ามีวิธีตั้งทีมวิ่งผลัดเป็นจำนวนคี่

6. รูปสามเหลี่ยมรูปหนึ่งมีเส้นรอบรูปยาว $2s$ ถ้าวงกลมแนบในมีรัศมี $r$ และระยะจากจุดศูนย์กลางวงกลมแนบในไปยังจุดยอดทั้งสามเป็น $s_a,s_b$ และ $s_c$ แล้วจงแสดงว่า
$$\frac{3}{4}+\frac{r}{s_a}+\frac{r}{s_b}+\frac{r}{s_c}\leq \frac{s^2}{12r^2}$$
----------
ปล. ผมว่าตั้งกระทู้ใหม่สำหรับโจทย์ สอวน. ครั้งนี้ดีไหมครับ จะได้เพิ่มลิงก์ในกระทู้ "รวมข้อสอบแข่งขัน ระดับมัธยมศึกษา" และจะได้เหมือนกับกระทู้ สอวน. ครั้งอื่น ๆ ด้วย
ตอบพร้อมอ้างอิงข้อความนี้
  #17  
Old 13 พฤษภาคม 2007, 22:11
M@gpie's Avatar
M@gpie M@gpie ไม่อยู่ในระบบ
ลมปราณไร้สภาพ
 
วันที่สมัครสมาชิก: 09 ตุลาคม 2003
ข้อความ: 1,227
M@gpie is on a distinguished road
Default

ข้อสอบยากจริงๆนะครับเนี่ย พี่ทำไม่ได้เลย

อ้างอิง:
ข้อความเดิมเขียนโดยคุณ Mathophile View Post
ช่วยเอาข้อสอบวิธีทำมาลงให้ครับ (โจทย์ยาวมาก... )
ปล. ผมว่าตั้งกระทู้ใหม่สำหรับโจทย์ สอวน. ครั้งนี้ดีไหมครับ จะได้เพิ่มลิงก์ในกระทู้ "รวมข้อสอบแข่งขัน ระดับมัธยมศึกษา" และจะได้เหมือนกับกระทู้ สอวน. ครั้งอื่น ๆ ด้วย
จัดการได้เลยครับ
__________________
PaTa PatA pAtA Pon!
ตอบพร้อมอ้างอิงข้อความนี้
  #18  
Old 13 พฤษภาคม 2007, 22:40
gon's Avatar
gon gon ไม่อยู่ในระบบ
ผู้พิทักษ์กฎขั้นสูง
 
วันที่สมัครสมาชิก: 29 มีนาคม 2001
ข้อความ: 4,608
gon is on a distinguished road
Thumbs up

ขอบคุณสำหรับข้อสอบที่พิมพ์มาทั้งหมดนะครับ.... อย่างไงก็ช่วยตรวจทานด้วยว่ามีพิมพ์ผิดหรือตกหล่นตรงไหนหรือไม่ จะได้ไม่ปวดหัวกันแบบไม่รู้จบ

ผมลองดู ข้อ 9. แบบเติมคำตอบ(วันแรก?) ตอนแรกพยายามแกะรอยว่าจะหาฟังก์ชันทั้งหมดที่สอดคล้อง
$$f(x^2+x+3) + 2f(x^2-3x+5) = 6x^2 - 10x + 7 \quad \quad\ (*)$$
ได้อย่างไร? ก็ยังไม่ออก คิดไปคิดมา นี่มันข้อเติมคำนี่นา โจทย์คงไม่ได้ต้องการอะไรที่มันลึกซึ้งมากมาย (หรือเปล่า ) ผมเลยโมเมว่า เอาละเอาแค่คำตอบชุดหนึ่งที่เป็นไปได้ละกัน

นั่นคือ ผมเลือกว่า f(x) มีผลเฉลยในรูปฟังก์ชันพหุนาม และเนื่องจากทางด้านขวามือเป็นพหุนามกำัลังสอง ดังนั้น f(x) ที่เป็นพหุนาม อย่างมากก็เป็นกำลังหนึ่ง นั่นคือ สมมติให้ f(x) = ax + b

เมื่อแทนในสมการ (*) แล้วเทียบสัมประสิทธิ์ปรากฎว่าโป๊ะเชะได้ a, b แบบไม่ขัดแย้งกันพอดี คือมีอยู่จริง จะได้ $f(x) = 2x - \frac{19}{3}$ ดังนั้น f(85) = 491/3

Note.ถ้าใครสามารถวิเคราะห์ได้ว่า มีหรือไม่มีฟังก์ชันอื่นที่สอดคล้องกับสมการเชิงฟังก์ชันดังกล่าวแล้ว ช่วยบอกทีนะครับ

ปล. ใครก็ได้ช่วยตั้งกระทู้และเอาข้อสอบไปแปะเป็นหัวข้อใหม่ก็ดีนะครับ. อันไหนที่เฉลยแล้วก็ทำลิงก์มากระทู้นี้ก็ได้

14 พฤษภาคม 2007 04:55 : ข้อความนี้ถูกแก้ไขแล้ว 2 ครั้ง, ครั้งล่าสุดโดยคุณ nongtum
เหตุผล: แก้เลขข้อ
ตอบพร้อมอ้างอิงข้อความนี้
  #19  
Old 14 พฤษภาคม 2007, 03:17
nooonuii nooonuii ไม่อยู่ในระบบ
ผู้พิทักษ์กฎทั่วไป
 
วันที่สมัครสมาชิก: 25 พฤษภาคม 2001
ข้อความ: 6,408
nooonuii is on a distinguished road
Default

ขอแปะไว้ซักสองข้อก่อนครับ ยากดีจริงๆ

วันแรก :

7. จากเอกลักษณ์

$(a+b+c)^2 = a^2+b^2+c^2 + 2(ab+bc+ca)$

$a^3+b^3+c^3 = 3abc + (a+b+c)(a^2+b^2+c^2-ab-bc-ca)$

เราจะได้ $ab+bc+ca = -\dfrac{1}{2}, abc = \dfrac{1}{6}$

ดังนั้น $a,b,c$ เป็นรากของพหุนาม $x^3 - x^2 - \dfrac{1}{2}x -\dfrac{1}{6}$

จึงเป็นรากของพหุนาม $x^4 - x^3 - \dfrac{1}{2}x^2 -\dfrac{1}{6}x$ ด้วย

ดังนั้น $a^4+b^4+c^4 = (a^3+b^3+c^3)+\dfrac{1}{2}(a^2+b^2+c^2)+\dfrac{1}{6}(a+b+c)=\dfrac{25}{6}$


8. สมการ $\displaystyle{\Big(\frac{1}{x}+1\Big)^{84}+7\Big(\frac{1}{x}+1\Big)-6=0 \Rightarrow 2x^{84}+91x^{83}+\cdots + 1 = 0}$

จะมี $\dfrac{1}{x_k-1},k=1,...,84$ เป็นราก

ดังนั้น $$\sum_{k=1}^{84}\frac{x_k}{x_k-1}=\sum_{k=1}^{84}\Big(1+\frac{1}{x_k-1}\Big)=84+\sum_{k=1}^{84}\frac{1}{x_k-1}=84-\frac{91}{2}=\frac{77}{2}$$
__________________
site:mathcenter.net คำค้น

14 พฤษภาคม 2007 08:50 : ข้อความนี้ถูกแก้ไขแล้ว 1 ครั้ง, ครั้งล่าสุดโดยคุณ nooonuii
ตอบพร้อมอ้างอิงข้อความนี้
  #20  
Old 14 พฤษภาคม 2007, 04:53
nongtum's Avatar
nongtum nongtum ไม่อยู่ในระบบ
ผู้พิทักษ์กฎทั่วไป
 
วันที่สมัครสมาชิก: 10 เมษายน 2005
ข้อความ: 3,246
nongtum is on a distinguished road
Default

ผมคิดว่าถ้าแค่จะให้มีลิงค์ในกระัทู้รวมลิงค์ ไม่ย้ายจะดีกว่าครับ แต่ถ้าอยากจะให้เกิดความเรียบร้อยผมก็ไม่ขัดนะ

ข้อ 16 หากมองว่า $n=\prod p_i^{k_i}$ มีจำนวนตัวประกอบ $\prod (k_1+1)$ ตัว ข้อนี้ก็จะตอบ $2^2\cdot3\cdot5\cdot7=420$ ครับ
__________________
คนไทยร่วมใจอย่าใช้ภาษาวิบัติ
ฝึกพิมพ์สัญลักษณ์สักนิด ชีวิต(คนตอบและคนถาม)จะง่ายขึ้นเยอะ (จริงๆนะ)

Stay Hungry. Stay Foolish.
ตอบพร้อมอ้างอิงข้อความนี้
  #21  
Old 14 พฤษภาคม 2007, 06:33
passer-by passer-by ไม่อยู่ในระบบ
ผู้พิทักษ์กฎทั่วไป
 
วันที่สมัครสมาชิก: 11 เมษายน 2005
ข้อความ: 1,442
passer-by is on a distinguished road
Smile

ข้อ 3 วันที่ 2 ก็คือข้อ 1 ในIMO 1995 ดีๆนี่เอง

ข้อ 8 คุณ nooonuii ลืมหารสองตอนท้ายๆนะครับ
__________________
เกษียณตัวเอง ปลายมิถุนายน 2557 แต่จะกลับมาเป็นครั้งคราว
ตอบพร้อมอ้างอิงข้อความนี้
  #22  
Old 14 พฤษภาคม 2007, 07:58
Art_ninja's Avatar
Art_ninja Art_ninja ไม่อยู่ในระบบ
จอมยุทธ์หน้าหยก
 
วันที่สมัครสมาชิก: 31 มีนาคม 2007
ข้อความ: 184
Art_ninja is on a distinguished road
Smile

แต่ข้อ 16 ผมได้ว่าจำนวนนั้นคือ 360 นะครับ
แล้วก็ข้อ 15 จะได้ว่า
$N\equiv 0(mod 9)...........(1)$
$N\equiv 2(mod 223)........(2)$ โดยที่ N คือก้อนยาวมากๆๆ ที่โจทย์ต้องการหาเศษเหลือ
จากทั้งสองสมการ ใช้ Chinese's reminder theorem แก้ออกมาจะได้ เศษคือ 225 ครับ
__________________
Defeat myself successfully is the most successful in my life...
ตอบพร้อมอ้างอิงข้อความนี้
  #23  
Old 14 พฤษภาคม 2007, 08:56
nongtum's Avatar
nongtum nongtum ไม่อยู่ในระบบ
ผู้พิทักษ์กฎทั่วไป
 
วันที่สมัครสมาชิก: 10 เมษายน 2005
ข้อความ: 3,246
nongtum is on a distinguished road
Default

ข้อ 16 คงตอบ 360 จริงๆครับ แต่จะแสดงอย่างไรว่ามันคือ 360

17. ให้ $p=n+30$ เทอมโจทย์จะกลายเป็น $p^2-p+11$ ซึ่งจะเป็นกำลังสองสมบูรณ์เมื่อ
$4(p^2-p+11)=(2p-1)^2+43=(2p-1)^2\pm2(2p-1)+1$
ซึ่งหมายถึง $p=11,-10$ ดังนั้นผลคูณที่หาคือ $(11-30)(-10-30)=760$
__________________
คนไทยร่วมใจอย่าใช้ภาษาวิบัติ
ฝึกพิมพ์สัญลักษณ์สักนิด ชีวิต(คนตอบและคนถาม)จะง่ายขึ้นเยอะ (จริงๆนะ)

Stay Hungry. Stay Foolish.
ตอบพร้อมอ้างอิงข้อความนี้
  #24  
Old 14 พฤษภาคม 2007, 09:44
nooonuii nooonuii ไม่อยู่ในระบบ
ผู้พิทักษ์กฎทั่วไป
 
วันที่สมัครสมาชิก: 25 พฤษภาคม 2001
ข้อความ: 6,408
nooonuii is on a distinguished road
Default

10. ให้ $z=e^{i\theta}$ เป็นรากของพหุนามดังกล่าว จะได้ว่า $e^{ni\theta}(\sqrt{3}e^{i\theta}-1)=1$

ใส่ค่าสัมบูรณ์ทั้งสองข้างจะได้

$|3e^{i\theta}-1|=1$

จัดรูปจะได้ $\cos{\theta}=\dfrac{\sqrt{3}}{2}$

ดังนั้น $\cos{2\theta}=2\cos^2{\theta}-1=\dfrac{1}{2}........(*)$

ต่อไปนำสมการ $\sqrt{3}z^{n+1}-z^n-1=0$ มาเทียบส่วนจริงกับส่วนจินตภาพ เราจะได้ระบบสมการ

$\sqrt{3}\cos{(n+1)\theta}-\cos{n\theta}=1$

$\sqrt{3}\sin{(n+1)\theta}-\sin{n\theta}=0$

กระจาย $\cos{(n+1)\theta}$ กับ $\sin{(n+1)\theta}$ แล้วจัดรูปใหม่จะได้

$\cos{n\theta}-2\sqrt{3}\sin{\theta}\sin{n\theta}=2 .........(1)$

$\sin{n\theta}+2\sqrt{3}\sin{\theta}\cos{n\theta}=0.........(2)$

$\cos{n\theta}\times (1) + \sin{n\theta}\times(2)$ จะได้

$\cos{n\theta}=\dfrac{1}{2}$

ดังนั้น $\cos{n\theta}=\cos{2\theta}$ จาก $(*)$

เราจึงได้ $n\theta = \pm 2\theta + 2k\pi $ เมื่อ $k$ เป็นจำนวนเต็ม

แต่จาก $(*)$ เราได้ $\theta = \pm\dfrac{\pi}{6}$

ดังนั้น $n = \pm 2 + 12k $

เราต้องการจำนวนนับ ดังนั้น $n=2,10,14,22,....$

จากการตรวจสอบพบว่า $n=2$ ใช้ไม่ได้ แต่ $n=10$ ใช้ได้

เพราะฉะนั้้น $n=10$ เป็นค่าน้อยที่สุด

ป.ล. ขอบคุณคุณ passer-by สำหรับข้อ 8 ครับ แก้แล้วเรียบร้อย
__________________
site:mathcenter.net คำค้น

14 พฤษภาคม 2007 13:04 : ข้อความนี้ถูกแก้ไขแล้ว 1 ครั้ง, ครั้งล่าสุดโดยคุณ nooonuii
ตอบพร้อมอ้างอิงข้อความนี้
  #25  
Old 14 พฤษภาคม 2007, 11:44
bon bon ไม่อยู่ในระบบ
สมาชิกใหม่
 
วันที่สมัครสมาชิก: 13 กุมภาพันธ์ 2006
ข้อความ: 6
bon is on a distinguished road
Default

อ้างอิง:
ข้อความเดิมเขียนโดยคุณ the__best__solution View Post
$\leq$ $\frac{3}{4}$ $+$ $\frac{x}{y+z}$ $+$ $\frac{y}{x+z}$ $+$ $\frac{z}{x+y}$
$\leq$ $\frac{3}{4}$ $+$ $\frac{1}{4}$ $\frac{y+z}{x}$ $+$ $\frac{x+z}{y}$ $+$ $\frac{x+y}{z}$
ขอถามพี่ๆหน่อยครับว่ามาได้ยังไง
ตอบพร้อมอ้างอิงข้อความนี้
  #26  
Old 14 พฤษภาคม 2007, 12:00
Art_ninja's Avatar
Art_ninja Art_ninja ไม่อยู่ในระบบ
จอมยุทธ์หน้าหยก
 
วันที่สมัครสมาชิก: 31 มีนาคม 2007
ข้อความ: 184
Art_ninja is on a distinguished road
Default

#24
ผมคิดแบบเดียวกับพี่ nooonuii ครับ แต่เพื่อนผมบอกว่าข้อนี้ตอบ n=10 อ่ะครับ(เป็นเฉลยของกรรมการด้วยนะครับ)
ช่วยแนะนำด้วยนะครับ
__________________
Defeat myself successfully is the most successful in my life...
ตอบพร้อมอ้างอิงข้อความนี้
  #27  
Old 14 พฤษภาคม 2007, 12:48
nooonuii nooonuii ไม่อยู่ในระบบ
ผู้พิทักษ์กฎทั่วไป
 
วันที่สมัครสมาชิก: 25 พฤษภาคม 2001
ข้อความ: 6,408
nooonuii is on a distinguished road
Default

ขออภัยครับ ลืมเช็คว่า n=2 ใช้ไม่ได้ครับ แก้ให้แล้วครับ
__________________
site:mathcenter.net คำค้น
ตอบพร้อมอ้างอิงข้อความนี้
  #28  
Old 14 พฤษภาคม 2007, 16:14
Art_ninja's Avatar
Art_ninja Art_ninja ไม่อยู่ในระบบ
จอมยุทธ์หน้าหยก
 
วันที่สมัครสมาชิก: 31 มีนาคม 2007
ข้อความ: 184
Art_ninja is on a distinguished road
Talking

ข้อ 12 นะครับ จากโจทย์ เราจะได้ว่าจะต้องสวมถุงเท้า 1 ก่อนรองเท้า 1 เสมอ
ดังนั้นเราจึงมองได้ว่า ถุงเท้า1$\rightarrow$ รองเท้า 1 กับ รองเท้า 1$\rightarrow$ ถุงเท้า1 เป็นส่วนย่อยเดียวกัน
ดังนั้นข้อนี้จึงตอบว่า
$$\frac{8!}{2!2!2!2!} = 2520 $$
__________________
Defeat myself successfully is the most successful in my life...

27 สิงหาคม 2007 06:57 : ข้อความนี้ถูกแก้ไขแล้ว 2 ครั้ง, ครั้งล่าสุดโดยคุณ Art_ninja
ตอบพร้อมอ้างอิงข้อความนี้
ตั้งหัวข้อใหม่ Reply


หัวข้อคล้ายคลึงกัน
หัวข้อ ผู้ตั้งหัวข้อ ห้อง คำตอบ ข้อความล่าสุด
ประกาศผลสอบ สอวน Final ศูนย์สวนกุหลาบ Coco ข่าวคราวแวดวง ม.ปลาย 8 06 มกราคม 2008 23:31
ข้อสอบ 4th TMO ณ ร.ร.เตรียมทหาร Mathophile ข้อสอบโอลิมปิก 20 14 มิถุนายน 2007 19:18
The First POSN-Mathematical Olympiad Rovers ข่าวคราวแวดวง ม.ปลาย 4 06 พฤษภาคม 2005 09:55
The First POSN-Mathematical Olympiad Rovers ปัญหาคณิตศาสตร์ทั่วไป 1 24 เมษายน 2005 02:12


กฎการส่งข้อความ
คุณ ไม่สามารถ ตั้งหัวข้อใหม่ได้
คุณ ไม่สามารถ ตอบหัวข้อได้
คุณ ไม่สามารถ แนบไฟล์และเอกสารได้
คุณ ไม่สามารถ แก้ไขข้อความของคุณเองได้

vB code is On
Smilies are On
[IMG] code is On
HTML code is Off
ทางลัดสู่ห้อง


เวลาที่แสดงทั้งหมด เป็นเวลาที่ประเทศไทย (GMT +7) ขณะนี้เป็นเวลา 22:34


Powered by vBulletin® Copyright ©2000 - 2024, Jelsoft Enterprises Ltd.
Modified by Jetsada Karnpracha